Friday, April 22, 2011

all 40 to 93

Q.3) Which management control practices, if followed, in performance measurement of investment centres are likely to induce goal congruence, in respect of following assets

a. (i) Idle (ii) Intangible (iii) Leased

b. (i) Cash (ii) Receivables (iii) Inventories

Ans. In some business units, the focus is on profit as measured by the difference between revenues and expenses. In other business units, profit is compared with the assets employed in earning it. We refer to the latter group of responsibility centers as investment centers.

Measuring Assets Employed

In deciding what investment base to use to evaluate investment center man­agers, headquarters asks two questions: First, what practices will induce busi­ness unit managers to use their assets most efficiently and to acquire the proper amount and kind of new assets? Presumably, when their profits are related to assets employed, business unit managers will try to improve their performance as measured in this way. \Senior management wants the actions that they take toward this end to be in the best interest of the whole corporation. Second, what practices best measure the performance of the unit as an economic entity?

Ø Cash

Most companies control cash centrally because central control permits use of a smaller cash balance than would be the case if each business unit held the cash balances it needed to weather the unevenness of its cash inflows and outflows. Business unit cash balances may well be only the "float" between daily receipts and daily disbursements. Consequently, the actual cash balances at the busi­ness unit level tend to be much smaller than would be required if the business unit were an independent company. Many companies therefore use a formula to calculate the cash to be included in the investment base. For example, Gen­eral Motors was reported to use 4.5 percent of annual sales; Du Pont was re­ported to use two months' costs of sales minus depreciation.

One reason to include cash at a higher amount than the balance carried by a business unit is that the higher amount is necessary to allow comparisons to outside companies. If only the actual cash were shown: by internal units would appear abnormally high and might mislead senior management.

Some companies omit cash from the investment base. These companies reason that the amount of cash approximates the current liabilities; if this is so, the sum of accounts receivable and inventories will approximate the working capital.

Ø Receivables

Business unit managers can influence the level of receivables, not only indirectly by their ability to generate sales, and directly, by establishing credit terms by approving individual credit accounts and credit limits, and by the collecting overdue amount. In the interest of simplicity, receivable included at the actual end-.of-period balances, although the average of intraperiod balances is conceptually a better measure of the am should be related to profits.

Whether to include accounts receivable at selling prices or at cost of goods sold is debatable. One could argue that the business unit's real investment in accounts receivable is only the cost of goods sold and that a satisfactory return on this investment is probably enough. On the other hand, it is possible to argue that the business unit could reinvest the money collected from accounts receivable, and, therefore, accounts receivable should be included at selling prices. The usual practice is to take the simpler alternative-that is, receivables at the book amount, which is the selling price less an allowance for bad debts.

If the business unit does not control credits and collections, receivables may be calculated on a formula basis. This formula should be consistent with the normal payment period-for example, 30 days' sales where payment is made 30 days after the shipment of goods.

Ø Inventories

Inventories ordinarily are treated in a manner similar to receivables –that is they are often recorded at end-of-period amounts even though intraperiod averages would be preferable conceptually. If the company uses LIFO (last in first out) for financial accounting purposes, a different valuation method usu­ally is used for business unit profit reporting because LIFO inventory bal­ances tend to be unrealistically low in periods of inflation. In these circumstances, inventories should be valued at standard or average costs, and these same costs should be used to measure cost of sales on the business unit income statement

If work-in-process inventory is financed by advance payments or by progress payments from the customer, as is typically the case with goods that require a long manufacturing period, these payments either are subtracted from the gross inventory amounts or reported as liabilities.

For e.g. with manufacturing periods a year or greater, Boeing received progress payments for its airplanes and recorded them as liabilities.

Some companies subtract accounts payable from inventory on the grounds that accounts payable represent financing of part of the inventory by ven­dors, at zero cost to the business unit. The corporate capital required for in­ventories is only the difference between the gross inventory amount and accounts payable. If the business unit can influence the payment period al­lowed by vendors, then including accounts payable in the calculation en­courages the manager to seek the most favorable terms. In times of high interest rates or credit stringency, managers might be encouraged to con­sider forgoing the cash discount to have, in effect, additional financing pro­vided by vendors. On the other hand, delaying payments unduly to reduce net current assets may not be in the company's best interest since this may hurt its credit rating.

Ø Leased Assets

Suppose the business unit whose financial statements are shown in Exhibit 1 (see page 21) sold its fixed assets for their book value of $300,000, returned the proceeds of the sale to corporate headquarters, and then leased back the assets at a rental rate of $60,000 per year. As Exhibit 2 (see page 21) shows, the business unit's in­come before taxes would decrease because the new rental expense would be higher than the depreciation charge that was eliminated. Nevertheless, eco­nomic valued added would increase because the higher cost would be more than offset by the decrease in the capital charge. Because of this, business unit managers are induced to lease, rather than own, assets whenever the in­terest charge that is built into the rental cost is less than the capital charge that is applied to the business unit's investment base. (Here, as elsewhere, this generalization oversimplifies because, in the real world, the impact of in­come taxes must also be taken into account.)

Many leases are financing arrangements-that is, they provide an alterna­tive way of getting to use assets that otherwise would be acquired by funds ob­tained from debt and equity financing. Financial leases (i.e., long-term leases equivalent to the present value of the stream of lease charges) are similar to debt and are so reported on the balance sheet. Financing decisions usually are made by corporate headquarters. For these reasons, restrictions usually are placed on the business unit manager's freedom to lease assets.

Ø Idle Assets

If a business unit has idle asset that can be used by other units, the business unit may be permitted to exclude them from the investment base if it classifies them as available. The purpose of this permission is to encourage business unit managers to release underutilized assets to units that may have better use for them. How­ever, if the fixed assets cannot be used by other units, permitting the business unit manager to remove them from the investment base could result in dys­functional actions For example; it could encourage the business unit manager to idle partially utilized assets that are not earning a return equal to the busi­ness unit's profit objective. If there is no alternative use for the equipment, any contribution from this equipment will improve company profits.

Ø Intangible Assets

Some companies tend to be R&D intensive (e.g., pharmaceutical firms such as Novartis spend huge amounts on developing new products); others tend to be marketing intensive (e.g., consumer products firms such as Unilever spend huge amounts on advertising). There are advantages to capitalizing intangi­ble assets such as R&D and marketing and then amortizing them over a se­lected life. This method should change how the business unit manager views these expenditures. By accounting for these assets as long-term investments, the business unit manager will gain less short-term benefit from reducing out­ lays on such item, For instance, if R&D expenditures are expensed immedi­ately, each dollar of R&D cut would be a dollar more in pretax profits. On the other hand, if R&D costs are capitalized, each dollar cut will reduce the assets employed by a dollar; the capital charge is thus reduced only by one dollar times the cost of capital, which has a much smaller positive impact on eco­nomic valued added.

SET.6

Q.1) What do you understand by Goal Congruence? What are the informal factors that influence goal congruence?

Ans: This term is used when the same goals are shared by top managers and their subordinates. This is one of the many criteria used to judge the performance of an accounting system. The system can achieve its goal more effectively and perform better when organizational goals can be well aligned with the personal and group goals of subordinates and superiors. The goals of the company should be the same as the goals of the individual business segments. Corporate goals can be communicated by budgets, organization charts, and job descriptions.

Ø Goal Congruence- Meaning Individuals work in different hierarchies and handle different responsibilities & may have different goals. But they must come together as far as Company’s Goal is concerned (there action must speak Co’s language.)

Goal Congruence

Example 1– The HR manager has devised a HR training program to enhance the skills of its sales personnel, with an objective to enhance their productivity But if company is in strategic need of attaining a certain sales volume in a given quarter, it can not do so on account of non availability of personnel.

Example 2– The marketing department has planned an impressive advertising campaign, which promises good returns, But say due to cash crunch Company’s current financial position may not let to lose the strings

Example 3 – Production Manager may get a good applause for reducing cycle time; But at what cost? Building up the high inventory i.e. higher investment in current assets. While doing so he just overlooked the financial interest of the company. • After completing the given activity in more efficient manner the concerned manager scores the point/s on his score card. • Whether his actions are leading to scoring of points on the organization’s score card too? if it is so then only one can say the organization is marching towards a common goal.

Every individual working in an organization has got his own motive to do the work. Individuals act in their own interest, based on their own motivations. And it is always not necessarily consistent with the Co’s goal. In a goal congruence process, the actions the people are led to take in accordance with their perceived self interest are also in the best interest of the organization i.e. Goal congruence ensures that the action of manager taken in their best interest is also in the best interest of the organization.

Informal factors that influence goal congruence:

Informal Factors –

External factors – set of attitudes of the society, work ethics of the society

Internal factors – (Factors within the organization)

· Culture-Common beliefs, shared values, norms of behavior & assumptions

· Implicitly accepted and explicitly built into.

· Mgt. Style – Informal/Formal

· The Communication Channels

· Perception and Communication – e.g. Budget (meaning) strict profit.

Ø Organizations with Business Divisions (Profit Centre) format have observed that Divisional Controllers experience divided loyalty in carrying out their functions, causing a possible dysfunction. How could such a situation be resolved? Define role of controller which suits your suggestion.

To the extent the decision are decentralized top management may lose some control. Relying on control reports is not as effective as personal knowledge of an operation. With profit center, top management must change its approach to control. Instead of personal direction senior management must rely to a considerable extent on management control reports.

Competent units that were once cooperating as functional units may now compete with one another dis advantageously. An increase in one manager’s profit may decrease those of another. This decrease in cooperation may manifest itself in a manager unwillingness to refer sales lead to another business unit, even though that unit is better qualified to follow up on the lead in production decision that have undesirable cost consequence on other units or in the hoarding of personnel or equipment that from the overall company standpoint would be better off used in another units.

There may be too much emphasis on short run profitability at the expense of long run profitability. In the desire to report high current profits, the profit center manager may skip on R&D, training, maintenance. This tendency is especially prevalent when the turnover of profit center managers is relatively high. In these circumstances, manager may have good reason to believe that their action may not affect profitability until after they have moved to other job.

There is no complete satisfactory system for ensuring that each profit center by optimizing its own profit , will optimize company profits.

If headquarter management is more capable or has better information then the average profit center manager the quality of some of the decision may be reduced.

Divisionalization may cause additional cost because it may require additional management staff personnel and recordkeeping and may lead to redundant at each profit center.

Business units as profit centers:

Business units are usually set up at profit centers. Business unit managers tend to control product development, manufacturing, and marketing resources. They are in a position to influence revenue and cost and as such can be held accountable for the bottom line. However as pointed out in the next section a business unit manager authority may be constrained such constrained should be incorporated in designing and operating profit center.

Constraint on business unit authority

To realize fully the advantage of the profit center concept the business unit manger would have to be as autonomous as the president of the independent company. As a practical matter however such autonomy is not feasible. If a company were divided into completely independent units the organization would be giving up the advantage of size and synergism. Also senior management authority that a board of director gives to the chief executive. Consequently business unit structure represents trade off between business unit autonomy and corporate constraint. The effectiveness of a business units organization is largely dependent on how well these trade off are made.

The performance of a profit center is appraised by comparing actual results for one or more orf these measures with budgeting amounts. In addition, data on competitors and the industry provide a good cross check on the appropriate of the budget. Data for individual companies are available from the securities and exchange commission for about key business ratios; standard & poor computer services, Inc; Robert Morris associates annual statement studies; and annual survey published in fortune, business week, and Forbes. Trade associations publish data for the companies in their industries.

Revenues: choosing the appropriate revenue recognition method is important. Should revenue be recognized at the time as order is received, at the time an order is shipped, or at the time cash is received?

In addition to that decision, issues related to common revenues may need to be considered. There are some situations in which two or more profit centers participate in the sales effort that results in a sale; ideally, each should be given appropriate credit for its part in this transaction. Many companies have not given much attention to the solution of these common revenue problems. They take the position that the identification of price responsibility for revenue generation is too complicated to be practical and that sale personnel must recognize they are working not only for their own profit center but also for the overall good of the company. They for example, may credit the business unit that takes an order for a product handled by the another unit with the equivalent of a brokerage commission or a finder fee. In the case of a bank the branch performing a service may be given explicit credit for that service even though the customer account is maintained in another branch.

Role of controller

· It should publish procedure and forms for the preparation of the budget.

· It should provide assistance to budgetees in the preparation of their budget.

· It should administer the process of making budget revision during the year.

· It should coordinate the work of budget departments in lower echelons

· It should analyze reported performance against budget, interprets the result, and prepares summary report for senior management.

Ø Part of a multinational group, Sundaram Shoe Company(SSC), established its own facilities in India over 75 years ago and enjoyed an excellent record-high market share for its diverse range of shoes, growth and profits. SC markets its products through company owned shops and its own personnel. Organization structure is functional. Since 2001, profitability, market share are slipping. Pressure from cheap Chinese shoes and also premium shoes like Nike has made the company think< of organizational restructuring and introducing Comensurate Control System to regain its position. Although SSC outsources, 30% of products, it is seen as a production oriented company. SSC wants to adopt measures to reduce costs, strengthen marketing and be in a position to produce and meet unexpected and unusual customer demands. How should the company reorganize to achieve Goal Congruence. Define Performance Metric?

In a goal congruent process, the actions people are led to take in accordance with their perceived self-interest are also in the best interest of the organization. A firm’s strategy has a major influence on its structure. The type of structure in turn influences the design of the organization’s management control systems. Sundaram Shoe Company’s (SSC) organization structure is functional which involves the notion of a manager who brings specialized knowledge to bear on decisions related to a specific function, vis-à-vis a general purpose manager who lacks the specialized knowledge. A skilled marketing and production manager would be able to make better decisions in their respective fields. He would also be able to supervise workers in the same function better than the generalist would. Thus an important advantage of the functional structure is efficiency. A major disadvantage of this structure is that there is no unambiguous way of determining the effectiveness of the separate functional managers because each function contributes jointly to the organization’s final output. Therefore, there is no way of determining how much of the profit was earned respectively by the several production departments.

Sundaram Shoe Company which was a market leader for a period of over 75 years has been losing market share, which has impacted its profitability. Also it needs to be seen that the company outsources about 30% of its products. The company aims to strengthen marketing, reduce costs and wants to be in a position to customize products as per the demands of the customer. Thus, Sundaram needs to re-organize its organization structure which is functional to a Business Unit form of organization. The benefits of the re-organization would be that the business unit or the division would be responsible for all the functions involved in producing and marketing a specified product line. The business managers act almost as if their units are separate companies. They are responsible for planning and co-coordinating the work of the separate functions. Their performance is measured by the profitability of the business unit. This is a valid criterion because profit reflects the activities of both marketing and production.

Though business unit managers exercise broad authority over their units, headquarters reserves certain key prerogatives. Headquarters are responsible for obtaining funds for the company as a whole and allocating it to the business unit, as well as approving budgets and judging the performance of business unit managers, setting their compensation.

A major advantage of the Business unit structure of organization is that because it is close to the market for its products than the headquarters, its manager may make sounder production and marketing decisions than headquarters might and the unit as a whole reacts to new threats or opportunities quickly. This re-organization would help in achieving goal congruence in the organization.

Performance Metrics are high-level measures what you are doing; that is, they assess your overall performance in the areas you are measuring. They are external in nature and are most closely tied to outputs, customer requirements, and business needs for the process.

The performance measurement system should cover the following areas at a minimum:

CUSTOMERS

  1. Performance against customer requirements
  2. Customer Satisfaction

PERFORMANCE OF INTERNAL WORK PROCESSES

  1. Cycle times
  2. Product and service quality
  3. Cost performance (could be productivity measures, inventory, etc.)

SUPPLIERS

  1. Performance of suppliers against your requirements

FINANCIAL

  1. Profitability (could be at the company, product line, or individual level)
  2. Market share growth and other standard financial measures

EMPLOYEE

  1. Associate satisfaction

SET .7

Q: 1) (A) Describe the factors which impact service organization

Ans: Factors which impact service organization:

Ø Absence of Inventory Buffer:

Goods can be held as inventory, which is a buffer that dampens the impact on production activity of fluctuations in sales volume. Services cannot be stored. The airplane seat, hotel room, hospital operating room, or the hours of lawyers, physicians, scientists, and other professionals that are not used today are gone forever. Thus, although a manufacturing company can earn revenue in the future from products that are on hand today, a service company cannot do so. It must try to minimize its unused capacity.

Moreover, the costs of many service organizations are essentially fixed in the short run. In the short run, a hotel cannot reduce its costs substantially by closing off some of its rooms. Accounting firms, law firms, and other professional organizations are reluctant to layoff professional personnel in times of low sales volume because of the effect on morale and the costs of rehiring and training.

Ø Difficulty in Controlling Quality:

A manufacturing company can inspect its products before they are shipped to the consumer, and their quality can be measured visually or with instruments (tolerances, purity, weight, color, and so on). A service company cannot judge product quality until the moment the service is rendered, and then the judgments are often subjective. Restaurant management can examine the food in the kitchen, but customer satisfaction depends to a considerable extent on the way it is served. The quality of education is so difficult to measure that few educational organizations have a formal quality control system.

Ø Labor Intensive:

Manufacturing companies add equipment and automate production lines, thereby replacing labor and reducing costs. Most service companies are labor intensive and cannot do this. Hospitals do add expensive equipment, but mostly to provide better treatment, and this increases costs. A law firm expands by adding partners and new support personnel.

Ø Multi-Unit Organizations:

Some service organizations operate many units in various locations; each unit relatively small. These organizations are fast-food restaurant chains, auto rental companies, gasoline service stations, and many others. Some of the units are owned; others operate under a franchise. The similarity of the separate units provides a common basis for analyzing budgets and evaluating performance not available to the manufacturing company. The information for each unit can be compared with system wide or regional averages, and high performers and low performers can be identified. However because units differ in the mix of services they provide, in the resources that they use, and in other ways, care must be taken in making such comparisons.

Q:1) (B) Explain special characteristics of professional organization which would have a bearing on their control system.

Ans: Special Characteristics of Professional Organization:

Ø Goals:

A dominant goal of a manufacturing company is to earn a satisfactory profit, specifically a satisfactory return on assets employed. A professional organization has relatively few tangible assets; its principal asset is the skill of its professional staff, which doesn't appear on its balance sheet. Return on assets employed, therefore, is essentially meaningless in such organizations. Their financial goal is to provide adequate compensation to the professionals.

In many organizations, a related goal is to increase their size. In part, this reflects the natural tendency to associate success with large size. In part, it reflects economies of scale in using the efforts of a central personnel staff and units responsible for keeping the organization up to- date. Large public accounting firms need to have enough local offices to enable them to audit clients who have facilities located throughout the world.

Ø Professionals:

Professional organizations are labor intensive, and the labor is of a special type. Many professionals prefer to work independently, rather than as part of a team. Professionals who are also managers tend to work only part time on management activities; senior partners in an accounting firm participate actively in audit engagements; senior partners in law firms have clients. Education for most professions does not include education in management, but quite naturally stresses the skills of the profession, rather than management; for this and other reasons, professionals tend to look down on managers. Professionals tend to give inadequate weight to the financial implications of their decisions; they want to do the best job they can, re- I regardless of its cost. This attitude affects the attitude of support staffs and nonprofessionals in the organization; it leads to inadequate cost control.

Ø Output and Input Measurement:

The output of a professional organization cannot be measured in physical terms, such as units, tons, or gallons. We can measure the number of hours a lawyer spends on a case, but this is a measure of input, not output. Output is the effectiveness of the lawyer's work, and this is not measured by the number of pages in a brief or the number of hours in the courtroom. We can measure the number of patients a physician treats in a day, and even classify these visits by type of complaint; but this is by no means equivalent to measuring the amount or quality of service the physician has provided. At most, what is measured is the physician's efficiency in treating patients, which is of some use in identifying slackers and hard workers. Revenues earned is one measure of output in some professional organizations, but these monetary amounts, at most, relate to the quantity of services rendered, not to their quality (although poor quality is reflected in reduced revenues in the long run).

Furthermore, the work done by many professionals is non repetitive. No two consulting jobs or research and development projects are quite the same. This makes it difficult to plan the time required for a task, to set reasonable standards for task performance, and to judge how satisfactory the performance was. Some tasks are essentially repetitive: the drafting of simple wills, deeds, sales contracts, and similar documents; the taking of a physical inventory by an auditor; and certain medical and surgical procedures. The development of standards for such tasks may be worthwhile, although in using these standards, unusual circumstances that affect a specific job must be taken into account.

Ø Small Size:

With a few exceptions, such as some law firms and accounting firms, professional organizations are relatively small and operate at a single location. Senior management in such organizations can personally observe what is going on and personally motivate employees. Thus, there is less need for a sophisticated management control system, with profit centers and formal performance reports. Nevertheless, even a small organization needs a budget, a regular comparison of performance against budget, and a way of relating compensation to performance.

Ø Marketing:

In a manufacturing company there is a clear dividing line between marketing activities and production activities; only senior management is concerned with both. Such a clean separation does not exist in most professional organizations. In some, such as law, medicine, and accounting, the profession's ethical code limits the amount and character of overt marketing efforts by professionals (although these restrictions have been relaxed in recent years). Marketing is an essential activity in almost all organizations, however. If it can't be conducted openly, it takes the form of personal contacts, speeches, articles, conversations on the golf course, and so on. These marketing activities are conducted by professionals, usually by professionals who spend much of their time in production work-that is, working for clients.

In this situation, it is difficult to assign appropriate credit to the person responsible for "selling" a new customer. In a consulting firm, for example, a new engagement may result from a conversation between a member of the firm and an acquaintance in a company, or from the reputation of one of the firm's professionals as an outgrowth of speeches or articles. Moreover, the professional who is responsible for obtaining the engagement may not be personally involved in carrying it out. Until fairly recently, these marketing contributions were rewarded subjectively- that is, they were taken into account in promotion and compensation decisions. Some organizations now give explicit credit, perhaps as a percentage of the project's revenue, if the person who "sold" the project can be identified.

Q:2) Every SBU is a profit center but every profit center is not a SBU? What are the conditions that should be fulfill for an organization unit to be converted into a profit center? What are the different ways to measure the performance of profit center? Discuss their relevant merits and demerits.

Ans: Conditions for an organization to be converted into a profit centre: Many management decisions involve proposals to increase expenses with the expectation of an even greater increase in sales revenue. Such decisions are said to involve expense/revenue trade-offs. Additional advertising expense is an example. Before it is safe to delegate such a trade-off decision to a lower-level manager, two conditions should exist.

· The manager should have access to the relevant information needed for making such a decision.

· There should be some way to measure the effectiveness of the trade-offs the manager has made.

A major step in creating profit centers is to determine the lowest point in an organization where these two conditions prevail. All responsibility centers fit into a continuum ranging from those that clearly should be profit centers to those that clearly should not. Management must decide whether the advantages of giving profit responsibility offset the disadvantages, which are discussed below. As with all management control system design choices, there is no clear line of demarcation.

Ø Ways to Measure Performance:

There are two types of profitability measurements used in evaluating a profit center, just as there are in evaluating an organization as a whole. First, there is the measure of management performance, which focuses on how well the manager is doing. This measure is used for planning, coordinating, and controlling the profit center's day-to-day activities and as a device for providing the proper motivation for its manager. Second, there is the measure of economic performance, which focuses on how well the profit center is doing as an economic entity. The messages conveyed by these two measures may be quite different from each other. For example, the management performance report for a branch store may show that the store's manager is doing an excellent job under the circumstances, while the economic performance report may indicate that because of economic and competitive conditions in its area the store is a losing proposition and should be closed. .

The necessary information for both purposes usually cannot be obtained from a single set of data. Because the management report is used frequently, while the economic report is prepared only on those occasions when economic decisions must be made, considerations relating to management performance measurement have first priority in systems design-that is, the system should be designed to measure management performance routinely, with economic information being derived from these performance reports as well as from other sources.

Ø Types of Profitability Measures

A profit center's economic performance is always measured by net income (i.e., the income remaining after all costs, including a fair share of the corporate overhead, have been allocated to the profit center). The performance of the profit center manager, however, may be evaluated by five different measures of profitability: (1) contribution margin, (2) direct profit, (3) controllable profit, (4) income before income taxes, or (5) net income

(1) Contribution Margin:

Contribution margin reflects the spread between revenue and variable expenses. The principal argument in favor of using it to measure the performance of profit center managers is that since fixed expenses are beyond their control, managers should focus their attention on maximizing contribution. The problem with this argument is that its premises are inaccurate; in fact, almost all fixed expenses are at least partially controllable by the manager, and some are entirely controllable. Many expense items are discretionary; that is, they can be changed at the discretion of the profit center manager. Presumably, senior management wants the profit center to keep these discretionary expenses in line with amounts agreed on in the budget formulation process. A focus on the contribution margin tends to direct attention away from this responsibility. Further, even if an expense, such as administrative salaries, cannot be changed in the short run, the profit center manager is still responsible for controlling employees' efficiency and productivity.

(2) Direct Profit:

This measure reflects a profit center's contribution to the general overhead and profit of the corporation. It incorporates all expenses either incurred by or directly traceable to the profit center, regardless of whether or not these items are within the profit center manager's control. Expenses incurred at headquarters, however, are not included in this calculation. A weakness of the direct profit measure is that it does not recognize the motivational benefit of charging headquarters costs.

(3) Controllable Profit:

Headquarters expenses can be divided into two categories: controllable and non controllable. The former category includes expenses that are controllable, at least to a degree, by the business unit manager-information technology services, for example. If these costs are included in the measurement system, profit will be what remains after the deduction of all expenses that may be influenced by the profit center manager. A major disadvantage of this measure is that because it excludes non controllable headquarters expenses it cannot be directly compared with either published data or trade association data reporting the profits of other companies in the industry.

(4) Income before Taxes:

In this measure, all corporate overhead is allocated to profit centers based on the relative amount of expense each profit center incurs. There are two arguments against such allocations. First, since the costs incurred by corporate staff departments such as finance, accounting, and human resource management are not controllable by profit center managers, these managers should not be held accountable for them. Second, it may be difficult to allocate corporate staff services in a manner that would properly reflect the amount of costs incurred by each profit center.

There are, however, three arguments in favor of incorporating a portion of corporate overhead into the profit centers' performance reports. First, corporate service units have a tendency to increase their power base and to enhance their own excellence without regard to their effect on the company as a whole. Allocating corporate overhead costs to profit centers increases the likelihood that profit center manager§ will question these costs, thus serving to keep head office spending in check. (Some companies have actually been known to sell their corporate jets because of complaints from profit center managers about the cost of these expensive items.) Second, the performance of each profit center will become more realistic and more readily comparable to the performance of competitors who pay for similar services. Finally, when managers know that their respective centers will not show a profit unless all-costs, including the allocated share of corporate overhead, are recovered, they are motivated to make optimum long-term marketing decisions as to pricing, product mix, and so forth, that will ultimately benefit (and even ensure the viability of) the company as a whole.

If profit centers are to be charged for a portion of corporate overhead, this item should be calculated on the basis of budgeted, rather than actual, costs, in which case the "budget" and "actual" columns in the profit center's performance report will show identical amounts for this particular item. This ensures that profit center managers will not complain about either the arbitrariness of the allocation or their lack of control over these costs, since their performance reports will show no variance in the overhead allocation. Instead, such variances would appear in the reports of the responsibility center that actually incurred these costs. .

(5) Net Income:

Here, companies measure the performance of domestic profit centers according to the bottom line, the amount of net income after income tax. There are two principal arguments against using this measure: (1) after tax income is often a constant percentage of the pretax income, in which case there would be no advantage in incorporating income taxes, and (2) since many of the decisions that affect income taxes are made at headquarters, it is not appropriate to judge profit center managers on the consequences of these decisions. There are situations, however, in which the effective income tax rate does vary among profit centers. For example, foreign subsidiaries or business units with foreign operations may have different effective income tax rates. In other cases, profit centers may influence income taxes through their installment credit policies, their decisions on acquiring or disposing of equipment, and their use of other generally accepted accounting procedures to distinguish gross income from taxable income. In these situations, it may be desirable to allocate income tax expenses

to profit centers not only to measure their economic profitability but also to motivate managers to minimize tax liability.

Merits:

  • The quality of decisions may improve because they are being made by managers closest to the point of decision.
  • The speed of operating decisions may be increased since they do not have to be referred to corporate headquarters. . Headquarters management, relieved of day-to-day decision making, can concentrate on broader issues.
  • Managers, subject to fewer corporate restraints, are freer to use their imagination and initiative.Because profit centers are similar to independent companies, they provide an excellent training ground for general management. Their managers gain experience in managing all functional areas, and upper management gains the opportunity to evaluate their potential for higher-level jobs.
  • Profit consciousness is enhanced since managers who are responsible' for profits will constantly seek ways to increase them. (A manager responsible for marketing activities, for example, will tend to authorize promotion expenditures that increase sales, whereas a manager responsible for profits will be motivated to make promotion expenditures that increase profits.).
  • Profit centers provide top management with ready-made information on the profitability of the company's individual components. . Because their output is so readily measured, profit centers are particularly responsive to pressures to improve their competitive performance.

Demerits:

· Decentralized decision making will force top management to rely more on management control reports than on personal knowledge of an operation, entailing some loss of control.

· If headquarters management is more capable or better informed than the average profit center manager, the quality of decisions made at the unit level may be reduced.

· Friction may increase because of arguments over the appropriate transfer price, the assignment of common costs, and the credit for revenues that were formerly generated jointly by two or more business units working together.

· Organization units that once cooperated as functional units may now be in competition with one another. An increase in profits for one manager may mean a decrease for another. In such situations, a manager may fail to refer sales leads to another business unit better qualified to pursue them; may hoard personnel or equipment that, from the overall company standpoint, would be better off used in another unit; or may make production decisions that have undesirable cost consequences for other units.

· Divisionalization may impose additional costs because of the additional management, staff personnel, and record keeping required, and may lead to task redundancies at each profit center.

Q:3) what are different types of Strategic Missions at SBU level? How do these missions affect Strategic Planning process and Budgeting at SBU Level?

Different Types of Strategic Missions:

Business Unit Mission:

In a diversified firm one of the important tasks of senior management is resource deployment, that is, make decisions regarding the use of the cash generated from some business units to finance growth in other business units. Several planning models have been developed to help corporate level managers of diversified firms to effectively allocate resources. These models suggest that a firm has business units in several categories, identified by their mission; the appropriate strategies for each category differ. Together, the several units make up a portfolio, the components of which differ as to their risk/reward characteristics just as the components of an investment portfolio differ. Both the corporate 'office and the business unit general manager are involved in identifying the missions of individual business units. Of the many planning models, two of the most widely used are Boston Consulting Group's two-by-two growth-share matrix and General Electric Company/McKinsey & Company's three-by-three industry attractiveness-business strength matrix. While these models differ in the methodologies they use to develop the most appropriate missions for the various business units, they have the same set of missions from which to choose: build, hold, harvest, and divest.

Ø Build: This mission implies an objective of increased market share, even at the expense of short-term earnings and cash flow (e.g., Merck's bio-technology, Black and Decker's handheld electric tools).

Ø Hold: This strategic mission is geared to the protection of the business unit's market share and competitive position (e.g.: IBM's mainframe computers).

Ø Harvest: This mission has the objective of maximizing short-term earnings and cash flow, even at the expense of market share (e.g., American Brands' tobacco products, General Electric's and Sylvania's light bulbs)

Ø Divest: This mission indicates a decision to withdraw from the business either through a process of slow liquidation or outright sale. While the planning models can aid in the formulation of missions, they are not cook books. A business unit's position on a planning grid should not be the sole basis for deciding its mission.

Ø Business Unit Competitive Advantage: Every business unit should develop a competitive advantage in order to accomplish its mission. Three interrelated questions have to be considered in developing the business unit's competitive= advantage. First, what is the structure of the industry in which the business unit operates? Second, how should the business unit exploit the industry's structure? Third, what will be the basis of the business unit's competitive advantage?

Ø Industry Analysis: Research has highlighted the important role industry conditions play in the performance of individual firms. Studies have shown that average industry profitability is, by far, the most significant predictor of firm performance. According to Porter, the structure of an industry should be analyzed in terms of the collective strength of five competitive forces.

1. The intensity of rivalry among existing competitors. Factors affecting direct rivalry are industry growth, product differentiability, number and diversity of competitors, level of fixed costs, intermittent overcapacity, and exit barriers.

2. The bargaining power of customers. Factors affecting buyer power are number of buyers, buyer's switching costs, buyer's ability to integrate backward, impact of the business unit's product on buyer's total costs, impact of the business unit's product on buyer's product quality/ performance, and significance of the business unit's volume to buyers.

3. The bargaining power of suppliers. Factors affecting supplier power are number of suppliers, supplier's ability to integrate forward, presence of substitute inputs, and importance of the business unit's volume to suppliers.

4. Threat from substitutes. Factors affecting substitute threat are relative price/performance of substitutes, buyer's switching costs, and buyer's propensity to substitute.

5. The threat of new entry. Factors affecting entry barriers are capital requirements, access to distribution channels, economies of scale, product differentiation, technological complexity of product or process, expected retaliation from existing firms, and government policy.

Ø We make three observations with regard to the industry analysis:

1. The more powerful the five forces are, the less profitable an industry is likely to be. In industries where average profitability is high (such as soft drinks and pharmaceuticals), the five forces are weak (e.g., in the soft drink industry, entry barriers are high). In industries where the average profitability is low (such as steel and coal), the five forces are strong (e.g., in the steel industry, threat from substitutes is high).

2. Depending on the relative strength of the five forces, the key strategic issues facing the business unit will differ from one industry to another.

3. Understanding the nature of each force helps the firm to formulate effective strategies. Supplier selection (a strategic issue) is aided by the analysis of the relative power of several supplier groups; the business unit should link with the supplier group for which it has the best competitive advantage. Similarly, analyzing the relative bargaining power of several buyer groups will facilitate selection of target customer segments.

Ø Generic Competitive Advantage:

The five-force analysis is the starting point for developing a competitive advantage since it helps to identify the opportunities and threats in the external environment. With this understanding, Porter claims that the business unit has two generic ways of responding to the opportunities in the external environment and developing a sustainable competitive advantage: low cost and differentiation.

Ø Low Cost: Cost leadership can be achieved through such approaches as economies of scale in production; experience curve effects, tight cost control, and cost minimization (in such areas as research and development, service, sales force, or advertising). Some firms following this strategy include Charles Schwab in discount brokerage, Wal-Mart in discount retailing, Texas Instruments in consumer electronics, Emerson Electric in electric motors, Hyundai in automobiles, Dell in computers, Black and Decker in machine tools, Nucor in steel, Lincoln Electric in arc welding equipment, and BIC in pens.

Ø Differentiation:

The primary focus of this strategy is to differentiate the product offering of the business unit, creating something that is perceived by customers as being unique. Approaches to product differentiation include brand loyalty (Coca-Cola and Pepsi Cola in soft drinks), superior customer service (Nordstrom in retailing), dealer network (Caterpillar Tractors in construction equipment), product design and product features (Hewlett-Packard in electronics), and technology (Cisco in communications infrastructure). Other examples of firms following a differentiation strategy include BMW in automobiles; Stouffer's in frozen foods, Neiman-Marcus in retailing, Mont Blanc in pens, and Rolex in wristwatches.

Ø Value Chain Analysis:

Business units can develop competitive advantage based on low cost, differentiation, or both. The most attractive competitive position is to achieve cost-cum-differentiation.

SET 8

1. What is a responsibility centre? List and explain different types of Responsibility Centers with sketches.

Responsibility centers:

A responsibility center is an organization unit that is headed by a manager who is responsible for its activities. In a sense, a company is a collection of responsibility centers. Each of which is represented by box on the on the organization are responsibility centers for section work shifts or other small organization units. At a higher level are departments or business units that consist of several of these smaller units plus staff and management people these larger units are also responsibility center. And from the stand point of senior management and the board of directors, the whole company is responsibility center although the term is usually used to refer to unit within the company.

Nature of responsibility centers

A responsibility center exist one or more purpose are its objectives. The company as a whole has goals, and senior management has decided on a set of strategies to accomplish these goals. The objectives of responsibility centers are to help implement these strategies. Because the organization is the sum of its responsibility centers, if the strategies are sound and if each responsibility center, if the strategies are sound and if each responsibility center meets its objectives the whole organization should achieve its goals. A responsibility center uses inputs, and a variety of services. Its work with these resources and it usually require working capital, equipment, and other asset to do this work. As a result of this work the responsibility center produces output which is classified either as goods if they are tangible or as services if they are intangible. Every responsibility center has output that is it does something. In a production plant, the outputs are goods. In staff units, such as human resources, transportation, engineering, accounting, and administration, the output s are services. For many responsibility centers, especially staff units, outputs are difficult to measure; nevertheless, they exist. The products produced by a responsibility center or to the outside marketplace. In the first case, the product are inputs to the other responsibility center in the latter case, they are output s of the whole organization.

Types of Responsibility Centers

Cost Center

Cost centers are divisions that add to the cost of the organization, but only indirectly add to the profit of the company. Typical examples include Research and Development, Marketing and Customer service. Companies may choose to classify business units as cost centers, profit centers, or investment centers. There are some significant advantages to classifying simple, straightforward divisions as cost centers, since cost is easy to measure. However, cost centers create incentives for managers to underfund their units in order to benefit themselves, and this underfunding may result in adverse consequences for the company as a whole (reduced sales because of bad customer service experiences, for example). Because the cost centre has a negative impact on profit (at least on the surface) it is a likely target for rollbacks and layoffs when budgets are cut. Operational decisions in a contact centre, for example, are typically driven by cost considerations. Financial investments in new equipment, technology and staff are often difficult to justify to management because indirect profitability is hard to translate to bottom-line figures. Business metrics are sometimes employed to quantify the benefits of a cost centre and relate costs and benefits to those of the organization as a whole. In a contact centre, for example, metrics such as average handle time, service level and cost per call are used in conjunction with other calculations to justify current or improved funding.

Profit Center

A responsibility centre is called a profit centre when the manager is held responsible for both costs (inputs) and revenues (outputs) and thus for profit. Despite the name, a profit centre can exist in nonprofits organizations (though it might not be referred to as such) when a responsibility centre receives revenues for its services. A profit centre is a big segment of activity for which both revenues and costs are accumulated: A centre, whose performance is measured in terms of both - the expense it incurs and revenue it earns, is termed as a profit centre. The output of a responsibility centre may either be meant for internal consumption or for outside customers. In the latter case, the revenue is realized when the sales are made. That is, when the output is meant for outsiders, then the revenue will be measured from the price charged from customers. If the output is meant for other responsibility centre, then management takes a decision whether to treat the centre as profit centre or not. In fact, any responsibility centre can be turned into a profit centre by determining a selling price for its outputs. For instance, in case of a process industry, the output of one process may be transferred to another process at a profit by taking into account the market price. Such transfers will give some profit to that responsibility centre. Although such transfers do not increase the Company’s assets, they help in management control process.

Investment Centre

An investment centre goes a step further than a profit centre does. Its success is measured not only by its income but also by relating that income to its invested capital, as in a ratio of income to the value of the capital employed. In practice, the term investment centre is not widely used. Instead, the term profit centre is used indiscriminately to describe centers that are always assigned responsibility for revenues and expenses, but may or may not be assigned responsibility for the capital investment. It is defined as a responsibility centre in which inputs are measured in terms of cost / expenses and outputs are measured in terms of revenues and in which assets employed are also measured. A responsibility centre is called an investment centre, when its manager is responsible for costs and revenues as well as for the investment in assets used by his centre. He is responsible for maintaining a satisfactory return on investment i.e. asset employed in his responsibility centre. The investment centre manager has control over revenues, expenses and the amounts invested in the centre’s assets. The manager of an investment centre is required to earn a satisfactory return. Thus, return on investment (ROI) is used as the performance evaluation criterion in an investment centre. He also formulates the credit policy, which has a direct influence on debt collection, and the inventory policy, which determines the investment in inventory. The Vice President (Investments) of a mutual funds company may be in charge of an Investment Centre. In the Investment Centre, the manager in charge is held responsible for the proper utilization of assets. He is expected to earn a satisfactory return on the assets employed in his responsibility centre. Measurement of assets employed poses many problems. It becomes difficult to determine the amount of assets employed in a particular responsibility centre. Some of the assets are in the physical possession of the responsibility centre while for some assets it may depend upon other responsibility centers or the Head Office of the company. This is particularly true of cash or heavy plant and equipment. Whether such assets should be included in the figure of assets employed of the responsibility centre and if included, at how much value, is a difficult question. On account of these difficulties, investment centers are generally used only for relatively large units, which have independent divisions, both manufacturing and marketing, for their individual products.

2. Explain the process of evaluation of Responsibility Center from one stage to another with the help of illustration-cum-experiences of the corporate.

Process of evaluation of Responsibility Center.

  1. The organization is divided into various responsibility centers. Each responsibility centre is put under the charge of a responsibility manager.

  1. The targets or budgets of each responsibility centre are set in consultation with the manager of responsibility centre, so that he may be able to give full information about his department. The manager of responsibility centre should know as what is expected of him - each centre should have a clear set of goals. The responsibility and authority of each centre should be well defined.

  1. Managers are charged with the items and responsibility, over which they can exercise a significant degree of direct control.

  1. Goals defined for each area of responsibility should be attainable with efficient and effective performance.

  1. The actual performance is communicated to the managers concerned. If it falls short of the standards, the variances are conveyed to the top management. The names of persons responsible for the variances are also conveyed so that responsibility may be fixed.

The purpose of all these steps is to assign responsibility to different individuals so that their performance is improved and costs are controlled. The personal factor in Responsibility Accounting is most important. The management may prepare the best plan or the budget and put up before its staff, but its success depends upon the initiative and the will of the workers to execute it

Example of Responsibility Center

The Sarva Shiksha Abhiyan emphasizes quality improvement in elementary education for which it deems necessary that resource groups and responsibility centers from national to sub-district levels are identified. These groups would oversee the policy, planning, implementation and monitoring of all quality related interventions. Their major role would be to advise and assist at various levels in curriculum development, pedagogical improvement, teacher education/training and activities related to classroom transaction. In order to facilitate a decentralized mode of education, these groups would need to be constituted at various operational levels, namely - national, state, district and sub district. The following could be involved in the groups:

National level - NCERT, NIEPA, Universities, NGOs, experts and eminent educationists.

State level - SCERT, SIEMAT, Universities, IASEs/CTEs, NGOs, experts and eminent educationists.

District level - DIETs, representatives from DPEP District Resource Group, higher educational institutions, innovative teachers from the districts, NGOs.

Sub-district - BRC/BEO, representatives from CRCs, innovative teachers.

3. Briefly define Discretionary Expense Center, Engineered Expense Center, Profit Centre and Investment Centre? How is budget prepared in Discretionary Expenses Centre?

Engineered expense centers:

Engineered expense center have the following characteristics:

- Their inputs can be measured in monetary terms.

- Their output can be measured in physical terms.

- The optimal dollar amount of input required to produce one unit of output can be established.

Engineered expense center usually are found in manufacturing operations. Warehousing, distribution, trucking and similar units in the marketing organization also may be engineered expense center and so many certain responsibility center within administrative and support department. Examples are accounts receivable account payable and payroll section in the controller department personnel record and cafeteria in the human resource department shareholder record in the corporate secretary department and the company motor pool. Such units perform repetitive task for which standard cost can be developed. In an engineered expense center the output multiplied by the standard cost of each unit produced represents what the finished product should have cost. When this cost is compared to actual costs, the difference between the two represents the efficiency of the organization unit being measured. We emphasize that engineered expense centers have other important tasks not measured by cast alone. The effectiveness of this aspect of performance should be controlled. For example expenses center supervisor are responsible for the quality of good and for the volume of production in addition to their responsibility for cost efficiency. Therefore the type and amount of production is prescribed and specific quality standards are set so that manufacturing costs are not minimized at the expense of quality. Moreover manager of engineered expense center may be responsible for activities such a training that are not related to current production judgment about their performance should include an appraisal of how well they carry out these responsibilities. There are few if any responsibility center in which all cost items are engineered. Even in highly automated production department the amount of indirect labor and of various services used can vary with management discretion. Thus, the term engineered costs center refers to responsibility center in which engineered cost predominate but in does not imply that valid engineering estimates can be made for each and every cost item.

Discretionary expense center:

The output of discretionary expenses center cannot be measured in monitory terms. They include administration and support units research and development organization and most marketing activities. The term discretionary does not mean that management judgment is capricious or haphazard. Management has decided on certain policies that should govern the operation of the company. Whether to match exceed or spend less than the marketing effort of its competitor; the level of service that the company provides to the customer. The appropriate amount of spending for R & D, financial planning public relation and many other activities. One company may have a small headquarter staff another company of similar size and in the same industry may have a staff that is 10 times as large. the management of both companies may be concerned that they made the correct decision on staff size but there is no objective way judging which decision was actually better manager are hired and paid to make such decision. After such a drastic change the level of discretionary expenses generally has a similar pattern from one year to the next. The difference between budgeted and actual expense is not a measure of efficiency in a discretionary expense center it is simply the difference between the budgeted input and the actual input. It in no way measures the value of the output. if actual expense do not exceed the budget amount, the manager has ‘lived within the budget ‘ however ,because by definition the budget does not purport to measure the optimum amount of spending we cannot say that living within the budgeted is efficient performance.

Profit Center

A responsibility centre is called a profit centre when the manager is held responsible for both costs (inputs) and revenues (outputs) and thus for profit. Despite the name, a profit centre can exist in nonprofits organizations (though it might not be referred to as such) when a responsibility centre receives revenues for its services. A profit centre is a big segment of activity for which both revenues and costs are accumulated: A centre, whose performance is measured in terms of both - the expense it incurs and revenue it earns, is termed as a profit centre. The output of a responsibility centre may either be meant for internal consumption or for outside customers. In the latter case, the revenue is realized when the sales are made. That is, when the output is meant for outsiders, then the revenue will be measured from the price charged from customers. If the output is meant for other responsibility centre, then management takes a decision whether to treat the centre as profit centre or not. In fact, any responsibility centre can be turned into a profit centre by determining a selling price for its outputs. For instance, in case of a process industry, the output of one process may be transferred to another process at a profit by taking into account the market price. Such transfers will give some profit to that responsibility centre. Although such transfers do not increase the Company’s assets, they help in management control process.

Investment Centre

An investment centre goes a step further than a profit centre does. Its success is measured not only by its income but also by relating that income to its invested capital, as in a ratio of income to the value of the capital employed. In practice, the term investment centre is not widely used. Instead, the term profit centre is used indiscriminately to describe centers that are always assigned responsibility for revenues and expenses, but may or may not be assigned responsibility for the capital investment. It is defined as a responsibility centre in which inputs are measured in terms of cost / expenses and outputs are measured in terms of revenues and in which assets employed are also measured. A responsibility centre is called an investment centre, when its manager is responsible for costs and revenues as well as for the investment in assets used by his centre. He is responsible for maintaining a satisfactory return on investment i.e. asset employed in his responsibility centre. The investment centre manager has control over revenues, expenses and the amounts invested in the centre’s assets. The manager of an investment centre is required to earn a satisfactory return. Thus, return on investment (ROI) is used as the performance evaluation criterion in an investment centre. He also formulates the credit policy, which has a direct influence on debt collection, and the inventory policy, which determines the investment in inventory. The Vice President (Investments) of a mutual funds company may be in charge of an Investment Centre. In the Investment Centre, the manager in charge is held responsible for the proper utilization of assets. He is expected to earn a satisfactory return on the assets employed in his responsibility centre.

Budget Preparation.

The decision that management make about a discretionary expense budget are different from the decisions that it makes about the budget for an engineered expense center. For the latter management decides whether the proposed operating budget represent the cost of performing task efficiently for the coming period. management is not so much concerned with the magnitude of the task because this is largely determined by the actions of other responsibility centers, such as the marketing departments ability to generate sales. In formulating the budget for a discretionary expense center, however management principal task is to decide on the magnitude of the job that should be done. These tasks can be divided generally into two types continuing and special. Continuing task are those that continue from year to year for example financial statement preparation by the controller’s office. Special tasks are one shot project for example developing and installing a profit budgeting system in a newly acquired division. The technique management by objective is often used in preparing the budget for a discretionary expense center. Management by objective is a formal process in which a budget purposes to accomplish specific tasks and state a mean for measuring whether these tasks have been accomplished. There are two different approach to planning for the discretionary expense center increment budgeting and zero based review.

SET .9

Q1. Describe inherent difficulties creation of profit centres may cause and advantages possible?

Under which situation creation of profit centre is not advisable.

Under which situation creation of profit centre is not advisable

Decentralized decision making will force top management to rely more on management con­trol reports than on personal knowledge of an operation, entailing some loss of control. If headquarters management is mere capable or better informed than the average profit center manager, the quality of decisions made at the unit level way be reduced. Friction may increase because of arguments over the appropriate transfer price, the assign­ment of common costs, and the credit for revenues that were formerly generated jointly by two or more business units working together.

Organization units that once cooperated as functional units may now be in competition with one another. An increase in profits for one manager may mean a decrease for another. In such situation a manager may fail to refer sales leads to another business unit better qualified to pursue them; may hoard personnel or equipment that, from the overall company’s, standpoint, would be better off used in another unit; or may make production decisions that have undesirable cost consequences for other units.

Divisionalization may impose additional costs because of the additional management, staff personnel, and record keeping required, and may lead to task redundancies at each profit center.Competent general managers may not exist in a functional organization because there may not have been sufficient opportunities for them to develop general management competence.

There may be too much emphasis on short-run profitability at the expense of long-run profitability. In the desire to report high current profits, the profit center manager may skimp on R&D, training programs, or maintenance. This tendency is especially prevalent when the turnover of profit center managers is relatively high. In these circumstances, managers may have good reason to believe that their actions may not affect profitability until after they have moved to other jobs. There is no completely satisfactory system for ensuring that optimizing the profits of each individual profit center will optimize the profits of the company as a whole.

Q2.What are the challenges faced in pricing corporate services provided to Business Units operating as “profit centers?”

Business Units as Profit Centers

Most business units are created as profit centers since managers in charge of such units typically control product development, manufacturing, and marketing resources. These managers are in a position to influence revenues and costs and as such can be held accountable for the "bottom line." However, as pointed out in the next section, a business unit manager's authority may be constrained in various ways, which ought to be reflected in a profit center's design and operation.

Constraints on Business Unit Authority

To realize fully the benefits of the profit center concept, the business unit manager would have to be as autonomous as the president of an independent company. As a practical matter, however, such autonomy is not feasible. If a company were divided into completely independent units, the organization would lose the advantages of size and synergy. Furthermore in delegating to business unit management all the authority that the board of directors has given to the CEO, senior management would be abdicating its own responsibility. Consequently, business unit structures represent trade-offs between business unit autonomy and corporate constraints. The effectiveness of a business unit organization is largely dependent on how well these trade-offs are made.

Constraints from Other Business Units.

One of the main problems occurs when business units must deal with one another. It is useful to think of managing a profit center in terms of control over three types of decisions:

(1) The product decision (what goods or services to make and sell),

(2) The marketing decision (how, where, and for how much are these goods or services to be sold?), and

(3) The procurement or sourcing decision (how to obtain or manufacture the goods or services). If a business unit manager controls all three activities, there is usually no difficulty in assigning profit responsibility and measuring performance. In general, the greater the degree of integration within a company,

the more difficult it becomes to assign responsibility to a single profit center for all three activities in a given product line; that is, if the production, procurement, and marketing decisions for a single product line are split among two or more business units, separating the contribution of each business unit to the overall success of the product line may be difficult.

Constraints from Corporate Management

The constraints imposed by corporate management can be grouped into three types:

(1) Those resulting from strategic considerations,

(2) Those resulting because uniformity is required, and

(3) Those resulting from the economies of centralization.

Most companies retain certain decisions, especially financial decisions, at the corporate level, at least for domestic activities. Consequently, one of the major constraints on business units results from corporate control over new investments. Business units must compete with one another for a share of the available funds. Thus, a business unit could find its expansion plans thwarted because another unit has convinced senior management that it has a more

Attractive program. Corporate management .also imposes other constraints. Each business unit has a "charter" that specifies the marketing and/or production activities that it is permitted to undertake, and it must refrain from operating beyond its charter, even though it sees profit opportunities in doing so. Also, the maintenance of the proper corporate image may require constraints on the quality of products or on public relations activities.

Companies impose some constraints on business units because of the necessity for Uniformity. One-constraint is that business Units must conform to corporate accounting and MCS This constraint is especially troublesome for units that have been acquired from another company and that have been accustomed to using different systems.

Q.3) Write Short Notes on

1. Zero Based Budgeting

2. Internal Control\

Zero Based Budgeting:

Zero-based budgeting is a technique of planning and decision-making which reverses the working process of traditional budgeting. In traditional incremental budgeting, departmental managers justify only increases over the previous year budget and what has been already spent is automatically sanctioned. No reference is made to the previous level of expenditure. By contrast, in zero-based budgeting, every department function is reviewed comprehensively and all expenditures must be approved, rather than only increases.[1] Zero-based budgeting requires the budget request be justified in complete detail by each division manager starting from the zero-base. The zero-base is indifferent to whether the total budget is increasing or decreasing.

The term "zero-based budgeting" is sometimes used in personal finance to describe the practice of budgeting every dollar of income received, and then adjusting some part of the budget downward for every other part that needs to be adjusted upward. It would be more technically correct to refer to this practice as "active-balanced budgeting".

Advantages of Zero-Based Budgeting:

  1. Efficient allocation of resources, as it is based on needs and benefits.
  2. Drives managers to find cost effective ways to improve operations.
  3. Detects inflated budgets.
  4. Municipal planning departments are exempt from this budgeting practice.
  5. Useful for service departments where the output is difficult to identify.
  6. Increases staff motivation by providing greater initiative and responsibility in decision-making.
  7. Increases communication and coordination within the organization.
  8. Identifies and eliminates wasteful and obsolete operations.
  9. Identifies opportunities for outsourcing.
  10. Forces cost centers to identify their mission and their relationship to overall goals.

Disadvantages of Zero-Based Budgeting:

  1. Difficult to define decision units and decision packages, as it is time-consuming and exhaustive.
  2. Forced to justify every detail related to expenditure. The R&D department is threatened whereas the production department benefits.
  3. Necessary to train managers. Zero-based budgeting must be clearly understood by managers at various levels to be successfully implemented. Difficult to administer and communicate the budgeting because more managers are involved in the process.
  4. In a large organization, the volume of forms may be so large that no one person could read it all. Compressing the information down to a usable size might remove critically important details.
  5. Honesty of the managers must be reliable and uniform. Any manager that exaggerates skews the results

Internal Control:

Internal control is defined as a process affected by an organization's structure, work and authority flows, people and management information systems, designed to help the organization accomplish specific goals or objectives.[1] It is a means by which an organization's resources are directed, monitored, and measured. It plays an important role in preventing and detecting fraud and protecting the organization's resources, both physical (e.g., machinery and property) and intangible (e.g., reputation or intellectual property such as trademarks). At the organizational level, internal control objectives relate to the reliability of financial reporting, timely feedback on the achievement of operational or strategic goals, and compliance with laws and regulations. At the specific transaction level, internal control refers to the actions taken to achieve a specific objective (e.g., how to ensure the organization's payments to third parties are for valid services rendered.) Internal control procedures reduce process variation, leading to more predictable outcomes

Describing Internal Controls:

Internal controls may be described in terms of: a) the objective they pertain to; and b) the nature of the control activity itself.

Objective categorization

Internal control activities are designed to provide reasonable assurance that particular objectives are achieved, or related progress understood. The specific target used to determine whether a control is operating effectively is called the control objective. Control objectives fall under several detailed categories; in financial auditing, they relate to particular financial statement assertions,[5] but broader frameworks are helpful to also capture operational and compliance aspects:

  1. Existence (Validity): Only valid or authorized transactions are processed (i.e., no invalid transactions)
  2. Occurrence (Cutoff): Transactions occurred during the correct period or were processed timely.
  3. Completeness: All transactions are processed that should be (i.e., no omissions)
  4. Valuation: Transactions are calculated using an appropriate methodology or are computationally accurate.
  5. Rights & Obligations: Assets represent the rights of the company, and liabilities its obligations, as of a given date.
  6. Presentation & Disclosure (Classification): Components of financial statements (or other reporting) are properly classified (by type or account) and described.
  7. Reasonableness-transactions or results appear reasonable relative to other data or trends.

Activity categorization

Control activities may also be described by the type or nature of activity. These include (but are not limited to):

  • Segregation of duties - separating authorization, custody, and record keeping roles to limit risk of fraud or error by one person.
  • Authorization of transactions - review of particular transactions by an appropriate person.
  • Retention of records - maintaining documentation to substantiate transactions.
  • Supervision or monitoring of operations - observation or review of ongoing operational activity.
  • Physical safeguards - usage of cameras, locks, physical barriers, etc. to protect property.
  • Analysis of results, periodic and regular operational reviews, metrics, and other key performance indicators (KPIs).
  • IT Security - usage of passwords, access logs, etc. to ensure access restricted to authorized personnel. S

Q4 .Veena Pvt. Ltd., a small multiproduct company is taken over by a multinational company ( e.g. Hindustan Lever.) What changes in the control system would you expect and why?

Since Veena is a small multiproduct company it would require changes in control system which would be related to transfer pricing a, as this company would generally provide inputs to HUL. Thus the domestic operations generally involve transfer of goods and services only In view of this difference many other considerations, in addition to the criteria used in domestic operations for the determination of transfer price, are involved. These include:

(a) Fair Price: This is an important factor one needs to consider while determining the transfer price for foreign operations. Companies that enter into joint ventures must ensure that the transfer price charged is fair. If such companies charge a higher transfer price, it would reduce the profits of the joint venture and as a result reduce the foreign partner's share of profits.

(b) Government Regulations: All countries have a regulatory framework under which business units operate. Where government rules and regulations regarding transfer prices are lenient, the parent company should fix a higher transfer price for all transfers to countries with high income tax rates. This approach would enable the parent company to minimize taxes in such countries.

(c) Exchange Control Restrictions: Every country has foreign exchange control regulations.

These regulations impose a limit on the amount of foreign exchange available for the import of certain goods. To accommodate the foreign subsidiary the parent company may have a lower transfer price so that the subsidiary is able to import a larger quantity of required goods.

(d) Income Tax Regulations: The rates of income tax vary from country to country. To overcome this difference the transfer price should be so fixed that countries with low tax rates show profits while others end up with a loss. This helps the parent company to reduce its taxes on a global basis.

(e) Desire to accumulate funds: A company that wishes to accumulate funds in a particular country may fix the transfer prices in such a manner that it facilitates shifting of funds into that country.

(I) Tariffs- and Duties: No country likes high imports. In order to restrict imports countries impose restrictions such as quantitative restrictions, high duties and tariffs and banning import of products. The general practice is to charge import duties as a percentage of the value of products imported, although a lower tariff may be levied if the import value is lower. It is seen that the impact of tariffs on the profitability of foreign operations is generally the reverse of the incidence of income taxes in transfer pricing. As such a low transfer price would lead to low import duties on transfer, the profit arising in that country would be high. This results in high income taxes in that country. It is therefore advisable that companies must compute the net effect of these factors while determining transfer prices.

In designing performance evaluation systems for acquired Veena company,HUL could use the following guidelines

Subsidiary managers should not be held responsible for translation effects. The simplest way to achieve this objective is to compare budgets and actual results using the same met­ric and isolate inflation-related effects through variance analysis. It is pointless for man­agers to worry about the appropriate metric. The MNE should choose whatever metric is more convenient.

Transaction effects are best handled through centralized coordination of the MNE's overall hedging needs. This is likely to be cheaper and simpler, and it prevents the subsidiary man­ager from becoming a foreign exchange rate forecaster and speculator.

The subsidiary manager should be held responsible for the dependence effects of exchange rates resulting from economic exposure.

Evaluation of the subsidiary as a basis for a decision to locate operations in a country or to relocate operations from a country should reflect the consequences of translation. Transaction and economic exposures.

SET 10

Q.1) What are the Special Characteristics of Professional Service Organization?

Answer:

Goals

A dominant goal of a manufacturing company is to earn a satisfactory profit, specifically a satisfactory return on assets employed. A professional organization has relatively few tangible assets; its principal asset is the skill of its professional staff, which 'doesn't appear on its balance sheet. Return on assets employed, therefore, is essentially meaningless in such organizations. Their financial goal is to provide adequate compensation to the professionals.

In many organizations, a related goal is to increase their size. In part, this reflects the natural tendency to associate success with large size. In part, it reflects economies of scale in using the efforts of a central personnel staff and units responsible for keeping the organization up-to-date. Large public accounting firms need to have enough local offices to enable them to audit clients who have facilities located throughout the world.

Professionals

Professional organizations are labor intensive, and the labor is of a special type. Many professionals prefer to work independently, rather than as part of a team. Professionals who are also managers tend to work only part time on management activities; senior partners in an accounting firm participate actively in audit engagements; senior partners in law firms have clients. Education for most professions does not include education in management, but quite naturally stresses the skills of the profession, rather than management; for this and other reasons, professionals tend to look down on managers.

Professionals tend to give inadequate weight to the financial implications of their decisions; they want to do the best job they can, regardless of its cost. This attitude affects the attitude of support staffs and nonprofessionals in the organization; it leads to inadequate cost control.

Output and Input Measurement

The output of a professional organization cannot be measured in physical terms, such as units, tons, or gallons. We can measure the number of hours a lawyer spends on a case, out this is a measure of input, not output. Output is the effectiveness of the lawyer's work, and this is not measured by the number of pages in a brief or the number of hours in the courtroom. We can measure the number of patients a physician Teats in a day, and even classify these visits by type of complaint; but this is by no means equivalent to measuring the amount or quality of service the physician has provided. At most, what is measured is tl1e physician's efficiency in treating patients, which is of some use in identifying slackers and hard workers. Revenues earned is one measure of output in some professional organizations, but these monetary amounts, at most, relate to the quantity of services rendered, not to their quality (although poor quality is reflected in reduced revenues in the long run).

Furthermore, the work done by many professionals is non repetitive. No two consulting jobs or research and development projects are quite the same. This makes it difficult to plan the time required for a task, to set reasonable standards for task performance, and to judge how satisfactory the performance was. Some tasks are essentially repetitive: the drafting of simple wills, deeds, sales contracts, and similar documents; the taking of a physical inventory by an auditor; and certain medical and surgical procedures. The development of standards for such tasks may be worthwhile, although in using these standards, usual circumstances that affect a specific job must be taken into account. Some professionals, notably scientists, engineers, and professors, are reluctant to keep track of how they spend their time, and this complicates the task of measuring performance. This reluctance seems to have its roots in tradition; usually, it can be overcome if senior management is willing to put appropriate emphasis on the necessity of accurate time reporting. Nevertheless, difficult problems arise in deciding how time should be charged to clients.

If the normal work week is 40 hours, should a job be charged for 1140th of a week's compensation for each hour spent on it? If so, how should work done on evenings and weekends be counted? (Professionals are "exempt" employees-that is, they are not subject to government requirements of overtime payments.) How to account for time spent reading literature, going to meetings, and otherwise keeping up-to-date?

Small Size

With a few exceptions, such as some law firms and accounting firms, professional organizations are relatively small and operate at a single location. Senior management in such organization can personally observe what is going on and personally motivate employees. Thus, there is less need for a sophisticated management control system, with profit centers and formal performance reports. Nevertheless, even a small organization needs a budget, a regular comparison of performance against budget, and a way of relating compensation to performance.

How is Marketing done in them?

In a manufacturing company there is a clear dividing line between marketing activities and production activities; only senior management is concerned with both. Such a clean separation does not exist in most professional organizations. In some, such as law, medicine, and accounting, the profession's ethical code limits the amount and character of overt marketing efforts by professionals (although these restrictions have been relaxed in recent years). Marketing is an essential activity in almost all organizations, however. If it can't be conducted openly, it takes the form of personal contacts, speeches, articles, conversations on the golf course, and so on. These marketing activities are conducted by professionals, usually by professionals who spend much of their time in production work-that is, working for clients.

In this situation, it is difficult to assign appropriate credit to the person responsible for "selling" a new customer. In a consulting firm, for example, a new engagement may result from a conversation between a member of the firm and an acquaintance in a company, or from the reputation of one of the professionals as an outgrowth of speeches or articles.

Moreover, the professional who is responsible for obtaining the engagement may not be personally involved in carrying it out. Until fairly recently, these marketing contributions were rewarded subjectively-that is, they were taken into account in promotion and compensation decisions. Some organizations now give explicit credit, perhaps as a percentage of the project's revenue, if the person who "sold" the project can be identified.

How do we evaluate the Performance Appraisal?

As noted earlier in regard to teachers, at the extremes the performance of professionals is easy to judge. Appraisal of the large percentage of professionals who are within the extremes is much more difficult. For some professions, objective measures of performance are sometimes unavailable: The recommendations of an investment analyst can be compared with actual market behavior of the securities; the accuracy of a surgeon's diagnosis can be verified by an examination of the tissue that was removed; and the doctors' skill can be measured by the success ratio of operations. These measures are, of course, subject to appropriate qualifications, and in most circumstances the assessment of performance is finally a matter of human judgment by superiors, peers, self, subordinates, and clients. Judgments made by superiors are the most common. For these, professional organizations increasingly use formal systems to collect performance appraisals as a basis for personnel decisions and for discussion with the professional. Some systems require numerical ratings of specified attributes of performance and provide for a weighted average of these ratings. Compensation may be tied, in part, to these numerical ratings. In a matrix organization, both the project leader and the head of the functional unit that is the professional's organizational "home" judge performance. "

Appraisals by a professional's peers, or by subordinates, are sometimes part of a formal control system. In some organizations, individuals may be asked to make a self-appraisal. Expressions of satisfaction or dissatisfaction from clients are also an important basis for judging performance, although such expressions may not always be readily forthcoming.

The budget can be used as the basis for measuring cost performance, and the actual time taken can be compared with the planned time. Budgeting and control of discretionary expenses are as important in a professional firm as in a manufacturing company.

Such financial measures are relatively unimportant in assessing a professional's contribution to the firm's, profitability, however. The professional's major contribution is related to quantity and above all quality of work, and its appraisal must be largely subjective. Furthermore, the appraisal must be made currently; it cannot wait until one learns whether a new building is well designed, a new control system actually works well, or a bond indenture has a flaw.

In some professions, internal audit procedures are used to control quality. In many accounting firms, the report of an audit is reviewed by a partner other than the one who is responsible for it, and the work of the whole firm is "peer reviewed" by another firm. The proposed design of a building may be reviewed by architects who are not actively involved in the project.

Q.2) What is a Non - Profit Organization? How is the performance of this organization evaluated?

Answer:

Introduction

A nonprofit organization, as defined by law, is an organization that cannot distribute assets or income to, or for the benefit of, its members, officers, or directors. The organization can, of course, compensate its employees, including officers and members, for services rendered and for goods supplied. This definition does not prohibit an organization from earning a profit; it prohibits only the distribution of profits. A nonprofit organization needs to earn a modest profit, on average, to provide funds for working capital and for possible “rainy days.”

Performance evaluation of nonprofit organization

For any organization, the most important reasons to measure performance are to improve effectiveness and to acquire information that will allow the organization to drive its agenda forward. If the motivation for doing evaluation remains outside an organization, the evaluation will have limited impact. To do performance assessment effectively, an organization must commit to adopting a culture of measurement, because acceptance must come from senior management, staff, funders, and board members alike.

Ø Board self-evaluation

Members of the Board of Directors should regularly evaluate the quality of their activities on a regular basis. Activities might include staffing the Board with new members, developing the members into well-trained and resourced members, discussing and debating topics to make wise decisions, and supervising the CEO. Probably the biggest problem with Board self-evaluation is that it does not occur frequently enough. As a result, Board members have no clear impression of how they are performing as members of a governing Board. Poor Board operations, when undetected, can adversely affect the entire organization.

Ø Staff and volunteer (individual) performance evaluation

Most of us are familiar with employee performance appraisals, which evaluate the quality of an individual’s performance in their position in the organization. Ideally, those appraisals reference the individual’s written job description and performance goals to assess the quality of the individual’s progress toward achieving the desired results described in those documents. Continued problems in individual performance often are the results of poor strategic planning, program planning and staff development. If overall planning is not done effectively, individuals can experience continued frustration, stress and low morale, resulting in their poor overall performance. Experienced leaders have learned that continued problems in performance are not always the result of a poor work ethic – the recurring problems may be the result of larger, more systemic problems in the organizations.

Ø Program evaluation

Program evaluations have become much more common, particularly because donors demand them to ensure that their investments are making a difference in their communities. Program evaluations are typically focused on the quality of the program’s process, goals or outcomes. An ineffective program evaluation process often is the result of poor program planning – programs should be designed so they can be evaluated. It can also be the result of improper training about evaluation. Sometimes, leaders do not realize that they have the responsibility to verify to the public that the nonprofit is indeed making a positive impact in the community. When program evaluations are not performed well, or at all, there is little feedback to the strategic and program planning activities. When strategic and program planning are done poorly, the entire organization is adversely effected.

Ø Evaluation of cross-functional processes

Cross-functional processes are those that span several systems, such as programs, functions and projects. Common examples of major processes include information technology systems and quality management of services. Because these cross-functional processes span so many areas of the organization, problems in these processes can be the result of any type of ineffective planning, development and operating activities.

Ø Organizational evaluation

Ongoing evaluation of the entire organization is a major responsibility of all leaders in the organization. Leaders sometimes do not recognize the ongoing activities of management to actually include organizational evaluations – but they do. The activities of organizational evaluation occur every day. However, those evaluations usually are not done systematically. As a result, useful evaluation information is not provided to the strategic and program planning processes. Consequently, both processes can be ineffective because they do not focus on improving the quality of operations in the workplace.

Q.3) A Well Diversified company – Pritam International Ltd. sells one of its divisions to a group of its own company managers. Explain what significant changes in systems and control procedures can be expected? Why?

Answer:

As, we Pritam International is a well diversified company. Sometimes, excessive diversification and that too in unrelated lines of business causes failure in the business operations. One of the major reason for failures of many Mergers and Diversification is excessive diversification. As, excessive diversification is ominous especially, in unrelated lines of business. As, there may be no advantage of operating synergy. Neither through:

I) Sharing common resources nor

II) Sharing common core competencies

Therefore, it may be a strategic decision by the promoters and directors of the company to sell one of its divisions. As, this may be impacting their core business. Sometimes, your core business tends to get neglected mainily due to excesive diversification. As, the division is being sold to its own company managers. There, might not be major changes in management control and systems. As, most of its managers will be the same. But, they will have more autonomy to take decisions independently after acquisition. Now there will be less red tapism and managers can take more risk. The managers will manage the firm in their own style. As, they are not answerable to their superiors.

Currently, they are answerable to their stake-holders. As, the management is completely in their hands and that too with full autonomy. The management might have identified the flaws in the previous controls and systems of the company because of which the company might not be so effective and efficient. As, they have been associated with the company over aperiod of time. They have a better understanding about the business dynamics and environment in which the firm operates. So, they can take necessary steps to overcome the flaws and improve the management control and systems.

So, that is why there will be some significant changes in the management control and systems and procedures if there is further scope for improvement.

SET 11

Q.1)Why Balance Score Card is considered superior to other methods of Performance Appraisal? Prepare Balance Score Card for any organization you are familiar with.

ANSWER:

What is the Balanced Scorecard?
The rationale for the development of the Balanced Scorecard was a growing dissatisfaction with traditional, financial measures of performance. These measures suffer from a number of serious drawbacks in that they take a short-term, lagged (i.e., historic) view of performance. The shift towards flexible, lean production/service systems in many firms has strengthened the requirement for performance measurement systems to become more broadly based, incorporating both non-financial and external measures of performance. According to Kaplan and Norton, the Balanced Scorecard provides a better assessment of performance as it "enables companies to track financial results while simultaneously monitoring progress in building the capabilities and acquiring the intangible assets they need for future growth".

The original scorecard designed by Kaplan and Norton contained four key groupings of performance measures. These four groupings, called ‘perspectives’ by Kaplan and Norton, were considered sufficient to track the key drivers of both current and future financial performance of the firm. The perspectives focused on the achievements of the firm in four areas: namely the financial, customer, internal business process and innovation/learning perspectives. The four perspectives can be represented as an interlinked hierarchy. The firm’s strategy underlies the whole scorecard, as the measures for each of the four perspectives are drawn from this strategy.

http://www.accaglobal.com/images/studentaccountant/12-99p34_pic1

To obtain a satisfactory overview of performance, the scorecard will require a mix of lagging and leading (forward looking) measures. Financial measures tend to be lagged and consequently, the measures chosen for the other perspectives will need to include leading measures. In general, outcome measures tend to be lagged, for example, current market share is the result of past decisions and consequently is a lagging measure. Thus the challenge in designing a Balanced Scorecard is to choose driver measures which lead changes in the outcome measures in the non-financial perspectives and which ultimately drive the financial measures.

Once the firm’s objectives have been agreed and the appropriate outcome and driver measures chosen for each of the perspectives, firm and managerial performance is assessed by comparing actual attainment on each measure with the target set for that measure.

Objective

Measure

Target

Actual

Benefits from adopting the Balanced Scorecard
There are several benefits from implementing a Balanced Scorecard. Originally the Balanced Scorecard was seen as a useful tool for performance measurement. In this role, the Balanced Scorecard was seen as integrating financial/non-financial, internal/external and leading /lagging information on firm performance in a coherent fashion.

Later it was realised that the Balanced Scorecard could play a pivotal role in the strategic management process. Because the Balanced Scorecard requires management to clarify and obtain consensus on the strategic objectives of the firm, it can assist in the communication of the chosen strategy, consequently aligning the efforts both of individuals and of departments. In this role, there is a clear link between the Balanced Scorecard and management by objectives (MBO). Effective implementation of a Balanced Scorecard project will generally involve the development of a series of hierarchical (cascaded) scorecards. Given the overall corporate scorecard, supporting scorecards can be developed for each department within the firm. Within each department, a scorecard can be developed for each manager (or perhaps even for each individual member of staff) which links the objectives on each perspective for that manager back to the objectives for each perspective outlined in the scorecard for the department and finally, back to the objectives listed in the firm’s overall scorecard.

The Balanced Scorecard could be used to assist in corporate restructuring. In recent years, many firms have migrated away from a traditional hierarchical structure to a flatter, team-based organisational structure. The Balanced Scorecard can support such changes, as it can help clarify the objectives and the critical success factors for the newly formed teams.

Apart from the communication and co-ordination roles of the Balanced Scorecard in strategic implementation, the Balanced Scorecard can be used to link strategy to specific critical success factors in the customer, internal business process and growth/learning perspectives. By setting both short and long-term targets for driver and outcome measures and by comparing actual attainment against target, feedback is obtained on how well the strategy is being implemented and on whether the strategy is working.

Building on the Balanced Scorecard’s use as a strategic management tool, it has been suggested that the Balanced Scorecard can play a role in the investment appraisal process(5). Traditional methods of investment appraisal such as discounted cash flow do not cope well with investments which generate indirect rather than direct financial returns. Examples of these include investments which enhance the future ‘flexibility’ of a firm or investments in the firm’s infrastructure, such as an enhanced management information system. The Balanced Scorecard can assist management’s investment appraisal decisions as it provides managers with a mechanism to incorporate the strategic aspects of the investment into the appraisal process. This could be achieved by using a weighting system developed from a firm’s Balanced Scorecard measures to evaluate new projects. An index score would be calculated for each investment opportunity and projects would then be ranked and selected based on this score.

Balance Score Card of Credit Card Company

Q .2)

Soniya Company has two Divisions: A & B. Return on Investment for both divisions is 20%. Details are given below:-

Particulars

Div A

Div B

Divisional sales

4000000

9600000

Divisional Investment

2000000

3200000

Profit

400000

640000

Analyse and comment on divisional performance of each.

ANSWER

As Profit Margin = Profit *100 Sales

Profit Margin for Division ‘A’= 4,00,000 /40,00,000 *100 = 10%

Profit Margin for Division ‘B’ = 6,40,000/ 96,00,000 *100 = 6.6%

Turnover of Investment = Sales * 100

Investment

Turnover of Investment for Division ‘A’ = 40,00,000/20,00,000 = 2 times

Turnover of Investment for Division ‘B’ = 96,00,000/32,00,000 = 3 times

As Return on investment for both Divisions A and B is 20%.

COMMENTS:-

Division ‘A’ – Although ‘A’ has more profit margin than Division ‘B’ that is 10% as compared to 6.6% of ‘B’, so it has more profitability but inspite of it, division ‘A’ has lower turnover of investment that its assets management is bad than Division ‘B’, it can be improved by increased sales or reducing investment.

Division ‘B’ – Needs to improve profit margin by increasing sales and reduce variable cost and sales at same price or by reducing salesprice and increase the volume of sales so that its profit would improve. As it has good assets management shown by its turnoverof Division ‘B’ that is 3 times which is better than Division ‘A’. So it can become profitable organisation by improving Profit Margin.

Q.4)Discuss and illustrate differences and similarities between

- Strategy Formulation and Management Control

- Management Control and Task Control

ANSWER

Some Distinction between Strategy Formulation and management Control

Characteristics

Strategy Formulation

Management Control

a) Focus of plan

On one aspect at a time

On entire organisation

b) Complexities

Many variables hence complex

Less complex

c) Nature of information

Tailor-made for the issue, more external and predictive, less accurate.

Integrated, more internal and historical, more accurate.

d) Structure

Unstructured and irregular, each problem being different

Rhythmic, definite pattern, set procedure

e) Communication of information

Relatively simple

Relatively difficult

f) Purpose of estimates

Show expected results

Lead to desired result

g) Persons involved

Staff and top management

Line and top management

h) No. of persons involved

Small

Large

i) Mental activity

Creative, analytical

Administrative, persuasive

j) Planning and control

Planning dominant but some control

Emphasis on both planning and control

k) Time horizon

Tends to be long

Tends to be short

l) End result

Policies and precedents

Action within policies laid

m) Appraisal of job done

Extremely difficult

Less difficult

b) Some Distinction between Management Control and Task Control

Characteristics

Task Control

Management control

a) Focus of plan

Single task or transaction

On entire organisation

b) Nature of information

Tailor-made to operation, specific, often non- financial, real time

Integrated, more internal and historical, more accurate

c) Persons involved

Supervisors

Line and top management

d) Mental activity

Follow directives or none as in case of machines or set objectives

Administrative, persuasive

e) Time horizon

Day to day

Tends to be short

f) Type of cost

Engineered- Existence of objective standard against which actuals can be compared makes control easier.

Discretionary- Control is more difficult due to subjective consideration.

SET 12

Q.2 Suresh Ltd. (Numerical) (MCS-2007)

(a) Define profit in this case and prepare a statement for both divisions and overall company.

Solution:

i) Profitability statement of Division A:-

Particulars

Amount(Rs.)

Selling price p.u.

35

Variable Cost p.u.

11

Contribution p.u.

24

Contribution p.u.

Expected sales (no. of units)

Total contribution

Total Fixed cost (Rs.)

Net profit (Rs.)

24

2000

48000

60000

(12000)

24

3000

72000

60000

12000

24

6000

144000

60000

84000

ii) Profitability statement of Division B:-

Selling p.u.

Total variable cost p.u.

Contribution p.u.

Expected sales (no. of units)

Total contribution

Total Fixed cost (Rs.)

Net profit (Rs.)

90

42

48

2000

96000

90000

6000

80

42

38

3000

114000

90000

24000

50

42

8

6000

48000

90000

(42000)

[Note: Total Variable cost p.u. = Variable cost p.u. (Rs.7) + Transfer price of intermediate product (Rs.35)]

iii) Profitability statement of Company as a whole:-

Expected sales

Net profit of division A (Rs.)

Net profit of Division B (Rs.)

Total Net profit

2000

(12000)

6000

(6000)

3000

12000

24000

36000

6000

84000

(42000)

42000

(b) State the selling price which maximizes profits for division B and company as a whole. Comment on why the latter price is unlikely to be selected by division B.

Solution:

As per the calculation in part (a), selling price p.u. of Rs.80 maximizes profit for division B whereas selling price p.u. of Rs.50 maximizes profit for the Company as a whole. However, if Division B opts for selling price p.u. of Rs.50 in order to maximize Company’s profit, it would suffer a loss of Rs.42000. Therefore, Division B would not select Selling price p.u. of Rs.50.




Q.3 Explain different organizational goals. Comment on goal of shareholder value maximization in particular.

Goals

Although we often refer to the goals of a corporation, a corporation does not have goals; it is an artificial being with no mind or decision-making ability of its own. Corporate goals are deter­mined by the chief executive officer (CEO) of the corporation, with the advice of other members of senior management, and they are usually ratified by the board of directors. In many corpo­rations, the goals originally set by the founder persist for generations. Examples are Henry Ford, Ford Motor Company; Alfred P. Sloan, General Motors Corporation; Walt Disney, Walt Disney Company; George Eastman, Eastman Kodak; and Sam Walton, Wal-Mart.

Economic Goals

Shareholder's value, Earning per share and Market value, all relate to maximizing shareholder's value, which is not a desirable goal, because what is 'maximum' is difficult to determine. Although optimizing shareholder value may be one goal, but there are other stakeholders in the business also such as customers, employees, creditors, community and so on. Again, shareholder value is usually equated with the market value of the company's stock. But market value is not an accurate measure of the worth of shareholders' investments. Besides, such value can be obtained only when the share is traded in the stock exchange.

It is interesting to note that Henry Ford's operating philosophy was 'satisfactory profit', not 'maximum profit'. He said, "A reasonable profit is right, but not too much. So, it has been my policy to force the price of the car down as fast as production would permit and give the benefit to the user and laborers, with resulting surprisingly enormous benefit to ourselves"

Other goals such as adding new products, or product-line or new business actually indicate normal organizational growth.

Social Goals

However, every organization has its share of responsibility towards the local community where it is situated, and the public at large. It is very difficult to incorporate in Management Control System such goals as taking pride in an organization which cares for the society and renders service to the public. Of course, any concrete structural programme indicating its operational expenses, methods of providing service, personnel involved in rendering service and the nature of the service in details can, however, be mentioned through an appropriate system.

Profitability

In a business, profitability is usually the most important goal.

Return on investment can be found by simply dividing profit (i.e., revenues minus expenses) by in­vestment, but this method does not draw attention to the two principal components: profit mar­gin and investment turnover.

In the basic form of this equation, "investment" refers to the shareholders' investment, which consists of proceeds from the issuance of stock, plus retained earnings.

One of manage­ment's responsibilities is to arrive at the right balance between the two main sources of fi­nancing: debt and equity. The shareholders' investment (i.e., equity) is the amount of financ­ing that was not obtained by debt, that is, by borrowing. For many purposes, the source of financing is not relevant; "investment" thus means the total of debt capital and equity capital.

"Profitability" refers to profits in the long run, rather than in the current quarter or year. Many current expenditure (e.g., amounts spent on advertising or research and development) reduce current profits but increase profits over time.

Some CEOs stress only part of the profitability equation. Jack Welch, former CEO of Gen­eral Electric Company, explicitly focused on revenue; he stated that General Electric should not be in any business in which its sales revenues were not the largest or the second largest of any company in that business. This does not imply that Welch neglected the other components

of the equation; rather, it suggests that in his mind there was a close correlation between mar­ket share and return on investment.

Other CEOs, however, emphasize revenues for a different reason: For them, company size is a goal. Such a priority can lead to problems. If expenses are too high, the profit margin will not give shareholders a good return on their investment. Even if the profit margin is satisfactory, the organization may still not earn a good return if the investment is too large.

Some CEOs focus on profit either as a monetary amount or as a percentage of revenue. This focus does not recognize the simple fact that if additional profits are obtained by a greater ­than proportional increase in investment, each dollar of investment has earned less.

Maximizing Shareholder Value

In the 1980s and 1990s the term shareholder value appeared frequently in the business liter­ature. This concept is that the appropriate goal of a for-profit corporation is to maximize share­holder value. Although the meaning of this term was not always clear, it probably refers to the market price of the corporation's stock. We believe, however, that achieving satisfactory profit is a better way of stating a corporation's goal, for two reasons.

First, "maximizing" implies that there is a way of finding the maximum amount that a com­pany can earn. This is not the case. In deciding between two courses of action, management usually selects the one it believes will increase profitability the most. But management rarely, if ever, identifies all the possible alternatives and their respective effects on profitability. Fur­thermore, profit maximization requires that marginal costs and a demand curve be calculated, and managers usually do not know what these are. If maximization were the goal, managers would spend every working hour (and many sleepless nights) thinking about endless alterna­tives for increasing profitability; life is generally considered to be too short to warrant such an effort.

Second, although optimizing shareholder value may be a major goal, it is by no means the only goal for most organizations. Certainly a business that does not earn a profit at least equal to its cost of capital is not doing its job; unless it does so, it cannot discharge any other respon­sibilities. But economic performance is not the sole responsibility of a business, nor is share­holder value. Most managers want to behave ethically, and most feel an obligation to other stakeholders in the organization in addition to shareholders.

Example: Henry Ford's operating philosophy was satisfactory profit, not maximum profit. He wrote let me say right here that I do not believe that we should make such an awful profit on our cars. A reasonable profit is right, but not too much. So it has been my policy to force the price of the car down as fast as production would permit, and give the benefits to the users and la­borers-with resulting surprisingly enormous benefits to ourselves.

By rejecting the maximization concept, we do not mean to question the validity of certain ob­vious principles. A course of action that decreases expenses without affecting another element, such as market share, is sound. So is a course of action that increases expenses with a greater­ than proportional increase in revenues, such as expanding the advertising budget. So, too, are actions that increase profit with a less than proportional increase in shareholder investment (or, of course, with no such increase at all), such as purchasing a cost-saving machine. These principles assume, in all cases, that the course of action is ethical and consistent with the cor­poration's other goals.

An organization's pursuit of profitability is affected by management's willingness to take risks. The degree of risk-taking varies with the personalities of individual managers. Never­theless there is always an upper limit; some organizations explicitly state that management's primary responsibility is to preserve the company's assets, with profitability considered a sec­ondary goal. The Asian .financial crisis during 1996-1998 is traceable, in large part, to the fact that banks in Asia's emerging markets made what appeared to be highly profitable loans with­out paying adequate attention to the level of risk involved.


Multiple Stakeholder Approach

Organizations participate in three markets: the capital market, the product market, and the factor market. A firm raises funds in the capital market, and the public stockholders are there­fore an important constituency. The firm sells its goods and services in the product market, and customers form a key constituency. It competes for resources such as human capital and raw materials in the factor market and the prime constituencies are the company's employees and suppliers and the various communities in which the resources and the company's operations are located.

The firm has a responsibility to all these multiple stakeholders-shareholders, customers, em­ployees, suppliers, and communities. Ideally, its management control system should identify the goals for each of these groups and develop scorecards to track performance.

Example: In 2005, the Acer Group, headquartered in Taiwan, was one of the largest computer companies The Company subscribed to the multiple stakeholder approach and managed its internal operations to satisfy the needs of several constituencies. To quote Stan 'Shih,-the founder, "The customer is number 1, the employee is number 2, the shareholder is number 3. I keep this mes­sage consistent with all my colleagues. I even consider the company's banks, suppliers, and others we do business with are our stakeholders; even society is stakeholder. I do my best to run the com­pany that way."

Lincoln Electric Company is well known for its philosophy that employee satisfaction was more important than shareholder value. James Lincoln wrote: "The last group to be considered is the stockholders who own stock because they think it will be more profitable than investing more in any other way. The absentee stockholder is not' of any value to the customer or to the worker, since he has no knowledge of nor interest in the company other than greater dividends and ad­vance in the price of his stock." Donald F. Hastings, chairman and chief executive officer, empha­sized that this was still the company's philosophy in 1996.

Q.4 Explain and illustrate with one example differences between 3 forms of internal audit- Financial, Operational & Management.

Financial Audit-

Financial Audit is a historically oriented, independent evaluation performed by

internal auditor or external auditor for the purpose of attesting to the fairness, accuracy and reliability of the financial data, providing protection for the entity's assets; evaluating the adequacy and accomplishment of the system (internal control) designed,

provide for the aforementioned Fairness and Protection, Financial data, while not being the only source of evidence, are the primary evidential source. The evaluation is

performed on a planned basis rather than a request".

Institute of Internal Auditor:-

Financial audit takes care of the protective aspect of the business and it does not

normally carry out constructive appraisal function of the business operations. It helps in detection and prevention of fraud. It also verifies whether documentation and flow of activities arc in conformity with the internal control system introduced and developed within the organization. It helps coordinating with statutory auditor to help them in proper discharge of their function. Besides, financial audit also ensures compliance with statutory laws especially in financial and accounting matters.

Objectives of Financial Audit:

-To see that established accounting systems and procedures have been complied with

-To see that proper records have been maintained for the fixed assets of the Concern to look into correctness of the financial data and records along with correctness of the accounting procedure followed.

-To see whether scrap, salvage and surplus materials have been properly accounted for etc.

-To see that internal control system has been working properly.

-To see that any abrupt variation in sales, purchases etc.; with respect to immediate previous year are not due to any irregularity

-To see that the credit control has been strictly followed.

-To see that all payments have been made with proper authorization and approval. .

-To see that preparation of salary and wage pay roll has been properly done.

budgetary control system, if any scope and performance of internal audit, if any, suggestions for improvements in performance, if any, and improved inventory policies.

The opinion expressed by the auditors shall be based on verified data, reference to ich shall also be made here and, if practicable, included after the company has been

forded on opportunity to comment on them.

Management Audit

It is a complex task closely related with the process of management. It is highly result oriented. It requires inter/multi-disciplinary approach as it involves examination, review and appraisal of various policies and actions of management on the basis of certain norms/standards.

It undertakes comprehensive and critical review of all organizational activities with wider perspective.

It goes beyond conventional audit and audits the efficacy of the management itself.

Definition:

It's a comprehensive and constructive examination of an organization, the structure of a company, institution or branch of government or of any components thereof, such as division or department and its plans, objectives, its means of operations and its use of human and physical facilities.

William P. Leonard

It's an investigation of a business from the higher level downwards in order to ascertain whether sound management prevails throughout, thus facilitating the most effective relationship with the outside world and the most efficient and smooth running internally.

Leslie Howard

It is an audit performed with the object of examining the efficacy of the institution/control systems, management procedures towards the achievement of enterprise goals.

Churchill & Cyert

It is an objective and independent appraisal of the effectiveness of managers and the effectiveness of the corporate structure in the achievement of company objectives and policies. Its aim is to identify existing and potential management weaknesses within an organization and to recommend ways to rectify these weaknesses.

Chartered Institute of Management Accountants London

Thus it can be seen that management audit is an examination, review and appraisal of the various policies and actions of the management. It is a tool for the evaluation of methods and performance in all the areas of the enterprise.

Objectives:

1. To ascertain the provision of proper control at different levels, their effectiveness I in accomplishing management goals.

2. Ascertain objectives of the organization are properly communicated and understood at all levels.

3. To reveal defects or irregularities in any of the elements examined and to indicate what improvements are possible to obtain the best results of the operations of the company.

4. To assist the management to achieve the most efficient administration of its operations.

5. To suggest to the management the ways and means to achieve the objectives if the management of the organization itself lacks the knowledge of efficient management.

6. It aims to achieve the efficiency of management and assess the strength and weaknesses of the organization structure, its management team and its corporate culture.

7. To ascertain the provision of proper control at different levels, their effectiveness in

accomplishing management goals.

8. Ascertain objectives of the organization are properly communicated and understood at all levels.

9. To reveal defects or irregularities in any of the elements examined and to indicate what improvements are possible to obtain the best results of the operations of the company.

10. To assist the management to achieve the most efficient administration of its operations.

11. To suggest to the management the ways and means to achieve the objectives if the

management of the organization itself lacks the knowledge of efficient management.

12. It aims to achieve the efficiency of management and assess the strength and weaknesses of the organization structure, its management team and its corporate culture.

13. To help the management at all levels in the effective and efficient discharge of their duties and responsibilities.

The auditor must apprise managerial performance at all levels of the organization. The audit starts right at the top level of the management. It studies the managerial performance at all the levels of management. The audit has to study the decision-making system of the organization and also the level of autonomy granted to the managers at different levels of the organization. The authority and responsibility given at the different levels of the management. One of the most important things that the audit must study is that the mangers at various levels use the authority.

Conducting Management Audit

Management audit requires an interdisciplinary approach since it involves a review of all aspects of management functions. It has to be conducted by a team of experts because this requires 3 varieties of skills, which one individual may not possess.

The team may consist of management experts, accountants, and the operation research specialists, the industry experts and even social scientists.

The auditors must have analytical mind and ability to look at a management function form the point of view of the organization as a whole. They therefore have to be properly trained in this aspect. They need to have through knowledge of the management science and they should be acquainted with the salient features of various functional areas.

Under financial audit, the entire emphasis is on macro-aspect, the individual transactions being- scrutinized for check of the aggregates. It is concerned with examination of transactions recorded in the books of account. It reviews the procedure and internal checks, and scrutinizes individual transactions for the purpose of verification, of Profit and Loss Account and Balance Sheet. Financial audit is not concerned with ~ avoidance of profiteering motive. It indicates the financial position and over~ performance of the business, regardless of its performance in various segments. Financial audit is applicable to all classes of companies and industries irrespective of size and Dan of operations.

Instead of serving the interest of the management and the Government, it serves interest of shareholders. Financial audit is organization - oriented. It is conducted under Sections 224 - 232 of the Companies Act 1956.

Financial Audit

Management Audit

It is concerned with financial aspects of

business transactions of the year under

audit

It is concerned with the review of the past

Performance to ascertain whether it is in tune with the objectives, policies and procedures of the enterprise.

The auditor examines the past financial

records to report his opinion on the truth

and fairness of the representations made in the financial statements. Examination of the performance of the management is

beyond his scope

The management auditor reports on performance of the management during a particular period and suggest ways to remedy the deficiencies, including modification of objectives, policies etc.

Past year '(Financial) transactions are

Covered Enterprises such as companies, trust and societies etc.

No limit as to the period to be covered

Financial audit is compulsory in the case of certain enterprises such as companies, trust and societies etc.

There is legal compulsion as regards management audit.

The auditor reports to the owner, i.e.

shareholders in the Case of a company

The auditor reports to the management

Q.5 Explain briefly various stages of management control process citing salient features of each.

Management control process involves communication of information to the managers at various levels of hierarchy and their interactions arising out of them. These communications aim towards attaining the organization's goals. But individual managers have their personal goals also. For example, a young manager with good education, experience, personality and social background joins a company like Britannia Industries or Reliance. The company finds him fit for the position as per job specifications, appoints him and makes him aware of what the company expects of him. The young manager sets his goals of gaining rich experience for his career progress besides adequate compensation packages. Naturally, his actions will be directed towards achieving his own objectives and goals while serving the company. Thus, his self-interest and the best interest of the organization are apparently in conflict. But the best results can be achieved by perfectly matching the two interests and this is called 'goal congruence'.

It is quite apparent that perfect congruence between the goals of the individual and the organization individual's goals and the organization's goals can never happen. Yet, the main purpose of a management control system is to assure goal congruence between the interest of the individual and the organization as far as practicable.

Management control systems

Formal and Informal Communication

As mentioned earlier, all the communication of information may be either formal or informal. The formal communication system involves strategic plan, budgets, standards and reports whereas the informal communication is made through letters and memos, verbally or even by facial expression.

Formal communications are all documented and addressed to the responsible managers for their information and actions, if necessary. However, the actions depend on the perception of the individual managers.

Informal communication, on the other hand, relates to some external factors-work ethics, management style and culture. Added to these factors is the existence of an informal organization within the structured formal organization.

Informality refers to the relaxation of sharp differentiation and explicit description of behavior as indicated in the hierarchy and thereby, moving away from superior/subordinate relationship. However, such relations depend on the personal capabilities of the manager such as education, experience, expertise, trust and cooperation. For example, Accounts Manager of Nasik Plant (see the organization chart in the diagram 3.2) reports to the General Manager of the Plant. While visiting the Corporate Office for attending a Training Course, he meets other colleagues, parallel officers and even the Finance Director. The latter communicates some important matter to him verbally and wants action thereon. Accounts Manager carried out the instructions so given. As per the organization chart, he should inform his General Manager, but it depends on his own perception of the situation, and he mayor may not report to the General Manager.

Work Ethics, Management Style and Culture

External factors like work ethics vary from place to place. Therefore, organization work culture depends on the general behavior of the people in the society where the organization situates. Work culture generally differs because of the life style and the attitude towards the work. For example, people of Mumbai lead very fast life. Time has more value at Mumbai as compared to Kolkata, where people take things easily and leisurely. Japanese and Korean people have reputation for their excellent work culture.

However, the most important internal factor is the organization's culture and climate. The culture refers to the set of common beliefs, attitudes, norms, relationships and assumptions that are explicitly or implicitly accepted and evidenced throughout the organization. The writer joined Union Carbide as an Assistant just three days before Christmas Eve. On the very second day, when he attended Christmas lunch, his table was shared by none other than the General Sales Manager Dr. W.R. Correa. He kept us amused with various stories of his recent tour abroad and recited Urdu 'shairies', even sharing jokes. Such a situation was unthinkable in Jessop & Co., where sharp differences were maintained at every level of hierarchy.

Management control systems

Climate is used to designate the quality of the internal environment that conditions the quality of cooperation, the development of individuals, the extent of members' dedication or commitment to organizational purpose and the efficiency with which that purpose is translated into results. Climate is the atmosphere in which individuals work help, judge, and reward, constrain and find out about each other. It influences moral-the attitude of the individual towards his/her work and environment.

Culture differs between the organizations, but cultural norms are extremely important. They are not written like formal communication. But the existence of a good culture can be felt from the behavior of the members of the organization. Once the writer landed up with his family at Hyderabad in the early morning to discover that nobody had come to receive them at the station. His visit was arranged through non other than the Director of the company himself. His unit being new, telephone directory did not include any number of his unit, but the parent organization's telephone number was located. When an executive of the parent company was contacted, he immediately sent an officer of the company with a car to pick us up to their Guest House, entertain with coffee and then put up in a Hotel. What subsequently happened is a different matter, but the attitude and treatment of that member of organization speak volumes about their excellent culture.

In any organization, the culture remains unchanged as long as the Chief Executive remains in position. When a new executive replaces him, there is likelihood of some change in the culture, unless the new Chief follows the footsteps of his predecessor and maintains it. Generally, if higher positions are filled in through promotion of internal executives, the culture remains unchanged and the traditions are maintained.

The other important internal factor which influences management control system is management style-that is the attitude of the superior to his subordinates and the latter's reaction through their perception of the attitude of their superiors. Again, the attitude ultimately stems from the temperament of the Chief Executive, who controls the entire organization. That is why R. W. Emerson said "an institute is the lengthened shadow of a man".

Importance of Informal Communication

An organization indulges in informal control process when encountering non-routine decision-­making or when seeking new information to increase understanding of some problem areas. During a very critical period in an organization, the writer found that the Chief Executive used to call managers informally at his residence or club to extract information in a relaxed manner rather than in a tense situation prevailing in the factory.

Formal Control Process

Formal communication system is structured as per the 'hierarchy outlined in the organization chart. The system has the following four components:

(a) Strategic plan and programme

(b) Budgeting

(c) Operations and measurement in responsibility centers (d) Reporting

(a) Strategic Plan and Programme

The foundation of management control process lies in the organization's goals and its strategies for attaining these goals. A strategic plan is prepared in order to implement the strategies, after carefully considering opportunities and threats in the external environment as well as the strengths and weaknesses in the internal environment. Thus, a strategic plan and programme is prepared as a guideline to budgeting.

(b) Budgeting

The strategic plan is converted to an annual budget incorporating planned expenditure and revenues for individual responsibility centers. Expenses and revenues are marked for each responsibility centre period wise, say monthly, quarterly, half yearly, and annually.

(c) Operations and Measurement

Responsibility centers operate within the framework of the budget, established standards, standing instructions, practices and operating procedures embodied in 'rules', and 'manuals'. Thus, besides budget, the responsibility centers are also guided by a large number of rules. They record the resources actually used and revenue earned. They also classify the data by programmes as well as by responsibility centers for performance measurement.

(d) Reporting

Actual performance is analyzed, measured and reported against plan, indicating variances and highlighting areas of weaknesses. If the performance is satisfactory, feedback information is sent to the responsibility centre concerned for praise or reward. If the same is unsatisfactory feedback communication is sent to the responsibility centre concerned for corrective action. If such action requires to be included in the budget, then the latter is revised to give effect to the changed position. If required, then the plan itself can be revised and a new basis of control may be established.

The aforesaid formal control process has been presented in the following diagram: